Proving Convergence of Absolute Value Sequence in Real Numbers

Click For Summary
For a sequence {an} converging to a in the real numbers, it is necessary to demonstrate that the sequence of absolute values {|an|} converges to |a|. The discussion explores using the inequality |an - a| ≤ ||an| - |a| to establish this convergence, considering three cases based on the sign of a: positive, negative, and zero. In each case, the goal is to find an appropriate N such that the conditions for convergence are satisfied. The reverse triangle inequality is mentioned as a potential tool, though concerns arise regarding its application due to the presence of double absolute values. Ultimately, the discussion emphasizes the importance of rigorously justifying the steps taken to prove the convergence of the absolute value sequence.
Daveyboy
Messages
57
Reaction score
0
For a sequence in the reals

{an} converges to a, show {|an|} converges to |a|.

For any e>0 the exists an N s.t. for any n>N |an-a|<e

I want to use this inequality, but there is something funny going on. I do not know how to justify it.

|an-a|\leq||an|-|a||
 
Physics news on Phys.org
Look at three separate cases.

1) a> 0. Can you show that, for some N, for all n> N a_n&gt; 0? (Take \epsilon= a/2.)

2) a< 0. Can you show that, for some N, for all n> N a_n&lt; 0?

3) a= 0. Here, ||a_n|- a|= ||a_n||= |a_n|.
 
Okay I see how to break it down case wise and find N accordingly. That will work nicely.

However, I was hoping to use the reverse triangle inequality but I run into the double abs. value. It just doesn't look right to say that
for any e>0 there exists and N s.t. for any n >N

|an - a| < e
and
|an - a| \geq |an| - |a|
implies

e>||an| - |a||

but if I showed this wouldn't it be true?
 
Question: A clock's minute hand has length 4 and its hour hand has length 3. What is the distance between the tips at the moment when it is increasing most rapidly?(Putnam Exam Question) Answer: Making assumption that both the hands moves at constant angular velocities, the answer is ## \sqrt{7} .## But don't you think this assumption is somewhat doubtful and wrong?

Similar threads

  • · Replies 5 ·
Replies
5
Views
2K
Replies
4
Views
2K
  • · Replies 13 ·
Replies
13
Views
2K
  • · Replies 2 ·
Replies
2
Views
2K
  • · Replies 1 ·
Replies
1
Views
2K
  • · Replies 5 ·
Replies
5
Views
2K
  • · Replies 7 ·
Replies
7
Views
2K
Replies
1
Views
1K
  • · Replies 4 ·
Replies
4
Views
2K
Replies
6
Views
3K